Вы находитесь на странице: 1из 18

Chapter # 2 Physics & Mathematics

Page # 1
manishkumarphysics.in
SOLVED EXAMPLES
1. Two vectors having equal magnitudes A make an angle u with each other. Find the magnitude and
direction of the resultant.
Sol. The magnitude of the resultant will be
B =
u + + cos AA 2 A A
2 2
= ) cos 1 ( A 2
2
u + =
2
cos A 4
2 2
u
= 2A cos
2
u
The resultant will make an angle o with the first vector where
tan o =
u +
u
cos A A
sin A
=
2
cos A 2
2
cos
2
sin A 2
2
u
u u
= tan
2
u
or, o =
2
u
.
Thus, the resultant of two equal vectors bisects the angle between them.
2. Two vectors of equal magnitude 5 unit have an angle 60 between them. Find the magnitude of (a) the
sum of the vectors and (b) the difference of the vectors.
B
A
+
B
A
B
A
+
B
A
60
B
120
Sol. Figure shows the construction of the sum B A

+ and the difference
B A

.
(a) B A

+ is the sum of
A

and
B

. Both have a magnitude of 5 unit and the angle between them is 60.
Thus the magnitude of the sum is
60 cos 5 5 2 5 5 | B A |
2 2
+ + = +

= 2 5 cos 30 = 3 5 unit.
(b)
B A

is the sum of
A

and (
B

). As shown in the figure, the anlge between


A

and (
B

) is 120.
The magnitudes of both
A

and (
B

) is 5 unit. So,
120 cos 5 5 2 5 5 | B A |
2 2
+ + =

= 2 5 cos 60 = 5 unit.
3. A force of 10.5 acts on a particle along a direction making an angle of 37 with the vertical. Find the
component of the force in the vertical direction.
Sol. The component of the force in the vertical direction will be
F
1
= F cos u = (10.5 N) cos 37)
= (10.5 N)
5
4
= 8.40 N
4. The work done by a force
F

during a displacement r

is given by
r F

. Suppose a force of 12 N acts on


a particle in vertically upward direction and the particle is displaced through 2.0 m in vertically downward
direction. Find the work done by the force during this displacement.
Chapter # 2 Physics & Mathematics
Page # 2
manishkumarphysics.in
Sol. The angle between the force
F

and the displacement r

is 180. Thus, the work done is


W =
r F

= Fr cos u
= (12 N) (2.0 m) (cos 180)
= 24 N m = 24 J.
5. The vector
A

has a magnitude of 5 unit,


B

has a magnitude of 6 unit and the cross product of


A

and
B

has a magnitude of 15 unit. Find the angle between


A

and
B

.
Sol. If the angle between
A

and
B

is u, the cross product will have a magnitude


|
A

| = AB sin u
or, 15 = 5 6 sin u
or, sin u =
2
1
Thus, u = 30 or, 150,
6. From the curve given in figure find
dx
dy
at x = 2, 6 and 10.
C
2 0 2 4 6 8 10 12 14
D
P
6-
4-
2-
u
1
u
2
Sol. The tangent to the curve at x = 2 is AC. It slope is tanu
1
=
BC
AB
=
4
5
Thus,
dx
dy
=
4
5
at x = 2
The tangent to the curve at x = 6 is parallel to the X-axis.
Thus,
dx
dy
= tan u = 0 at x = 6.
The tangent to the curve at x = 10 is DF. Its slope is tan u
2
=
EF
DE
=
4
5
.
Thus,
dx
dy
=
4
5
at x = 10.
7. Find
dx
dy
if y = e
x
sin x
Sol. y = e
x
sin x
So
dx
dy
=
dx
d
(e
x
sin x) = e
x
dx
d
(sin x) + sin x
dx
d
(e
x
)
e
x
cos x + e
x
sin x = e
x
(cos x + sin x).
8. The height reached in time t by a particle thrown upward with a speed u is given by
h = ut
2
1
gt
2
where g = 9.8 m/s
2
is a constant. Find the time taken in reaching the maximum height.
Chapter # 2 Physics & Mathematics
Page # 3
manishkumarphysics.in
Sol. The height h is a function of time. Thus, h will be maximum when
dt
dh
= 0. We have,
h = ut =
2
1
gt
2
or,
dt
d
dt
dh
=
(ut)
dt
d
|
.
|

\
|
2
gt
2
1
= u
dt
dt

2
1
g
dt
d
(t
2
)
= u
2
1
g (2t) = u gt.
For maximum h,
dt
dh
= 0
or, u gt = 0 or, t =
g
u
.
9. Evaluate
}
+ +
6
3
2
dx ) 5 x 3 x 2 ( .
Sol.
}
+ + dx ) 5 x 3 x 2 (
2
=
} } }
+ + dx 5 dx x 3 dx x 2
2
=
} } }
+ + dx x 5 dx x 3 dx x 2
0 2
=
1
x
5
2
x
3
3
x
2
2 3
+ +
=
x 5 x
2
3
x
3
2
2 3
+ +
Thus,
}
+ +
6
3
2
dx ) 5 x 3 x 2 ( =
6
3
2 3
x 5 x
2
3
x
3
2
(

+ +
=
3
2
(216 27) +
3
2
(36 9)+ 5(6 3)
= 126 + 40.5 +15 = 181.5.
10. Round off the following numbers to three significant digits (a) 15462, (b) 14.745, (c) 14.750 and (d)
14.650 10
12
.
Sol. (a) The third significant digit is 4. This digit is to be rounded. The digit next to it is 6 which is greater
than 5. The third digit should, therefore, be increasesd by 1. The digits to be dropped should be
replaced by zeroes because they appear to the left of the decimal. Thus, 15462 becomes 15500 on
rounding to three significant digits.
(b) The third significant digit in 14.745 is 7. The number next to it is less than 5. So 14.745 becomes
14.7 on rounding to three significant digits.
(c) 14.750 will become 14.8 because the digit to be rounded is odd and the digit next to it is 5.
(d) 14.650 10
12
will become 14.6 10
12
becausethe digit to be rounded is even and the digit next to it
is 5.
11. Evaluate
3 . 33
1374 2 . 25
. All the digits in this expression are significant.
Chapter # 2 Physics & Mathematics
Page # 4
manishkumarphysics.in
Sol. We have
3 . 33
1374 2 . 25
= 1039.7838...
Out of the three numbers given in the expression 25.2 and 33.3 have 3 significant digits and 1374 has
four. The answer should have three significant digits, it becomes 1040. Thus, we write
3 . 33
1374 2 . 25
= 1040.
Ex. Evaluate 24.36 + 0.0623 + 256.2
Sol. 24.36
0. 0623
256.2

280.7
The sum is 280.7.
13. The focal length of a concave mirror obtained by a student in repeated experiments are given below.
Find the average focal lengthwith uncertainty in o limit.
No. of observation focal length in cm
1 25.4
2 25.2
3 26.6
4 25.1
5 25.2
6 25.5
7 25.4
8 25.4
9 25.3
10 25.7
Sol. The average focal length

=
=
10
1 i
i
f
10
1
f

= 25.37 = 25.4
The calculation of o is shown in the table below :
i f
i
cm f
i
cm
(f
1
)
2
cm
2
cm
2
1 25.4 0.0 0.00
2 25.2 -0.2 0.04
3 25.6 0.2 0.04
4 25.1 0.3 0.09
5 25.3 -0.1 0.01
6 25.2 -0.2 0.04
7 25.5 0.1 0.01
8 25.4 0.0 0.00
9 25.3 -0.1 0.01
10 25.7 0.3 0.09
0.33
2
1
) f f (

E
2 2
1
cm 033 . 0 ) f f (
10
1
= E = o

= 0.18 cm.
= 0.2 cm.
Thus, the focal length is likely to be within (25.4 0.2 cm) and we write
f = (25.4 0.2) cm.
Chapter # 2 Physics & Mathematics
Page # 5
manishkumarphysics.in
QUESTIONS FOR SHORT ANSWER
1. Is a vector necessarily changed if it is rotated through an angle?
2. Is it possible to add two vectors of unequal magnitudes and get zero? It is possible to add three vectors of
equal magnitudes and get zero?
3. Does the phrase "direction of zero vector" have physical significance? Discuss in terms of velocity, force etc.
4. Can you add three unit vectors to get a unit vector? Does your answer change if two unit vectors are along the
co-ordinate axes?
5. Can we have physical quantities having magnitude and direction which are not vectors?
6. Which of the following two statements is more appropriate?
(a) Two forces are added using triangle rule because force is a vector quantity.
(b) Force is a vector quantity because two forces are added using triangle rule.
7. Can you add two vectors representing physical quantities having different dimensions? Can you multiply two
vectors representing physical quantities having different dimensions?
8. Can a vector have zero component along a line and still have nonzero magnitude ?
9. Let c
1
and c
2
be the angles made by
A

and
A

with the positive X-axis. Showthat tanc


1
= tanc
2
. Thus, giving
tanc does not uniquely determine the direction of A

.
10. Is the vector sum of the unit vectors
i

and j

a unit vector? If no, can you multiply this sum by a scalar


number to get a unit vector?
11. Let j

4 i

3 A + =

. Write four vectors


B

such that
B A

=
but A = B.
12. Can you have
B A B A

=
with A= 0 and B = 0? What if one of the two vectors is zero?
13. If 0 B A =

, can you say that (a)
B A

=
, (b)
B A

=
?
14. Let j 4 i 5 A

= and j 6 i 5 . 7 B

+ = . Do we have A k B

= ? Can we say k
A
B
=

?
Objective - I
1. A vector is not changed if
(A) it is rotated through an arbitary angle (B) it is multiplied by an arbitary scalar
(C) it is cross multiplied by a unit vector (D*) it is slid parallel to itself
+i: =l<zi ll-- -ti ti-i t. l<
(A) :=+i l+=i - l-z+ +i i = inii i
(B) :=+i l+=i - l-z+ ~l<zi ilzi = n ii l+i i
(C) :=+i l+=i (+i+ =l<zi = ni- l+i i
(D) :=+i - + =ni-i-- l-iil- l+i i
2. Which of the sets given belowmay represent the magnitudes of three vectors adding to zero ?
(A) 2, 4, 8 (B) 4, 8, 16 (C*) 1, 2, 1 (D) 0.5, 1, 2
l--- n = +i-=i =n - +- +-i t l+ :=+ -i- =l<zii + i n +i liini zi - i-- tini
(A) 2, 4, 8 (B) 4, 8, 16 (C) 1, 2, 1 (D) 0.5, 1, 2
Chapter # 2 Physics & Mathematics
Page # 6
manishkumarphysics.in
3. The result of

A and

B
makes an angle o with

A
and | with

B
,
(A) o < | (B) o < | if A < B (C*) o < | if A > B (D) o < | if A = B
A

-ii
B

+i liini.
A

= o +i i -ii
B

= | +i i -i-i t
(A) o < | (B) o < | l< A< B (C) o < | l< A> B (D) o > | l< A= B
4. The component of a vector is
(A) always less than its magnitude (B) always greater than its magnitude
(C) always equal to its magnitude (D*) none of these
l+=i =l<zi +i i+
(A) =< :=+ lnii = +n ti-i t (B) =< :=+ lnii = ~li+ ti -i t
(C) =< :=+ lnii + i ti -i t (D) :-n = +i: -ti
5. A vector

A
points vertically upward and

B
points towards north. The vector product

A
x

B
is
(A*) along west (B) along east (C) zero (D) vertically downward
=l<zi
A

-i i - +i ~i :ln- t -ii
B

--i +i ~i =l<zi ni- B x A



t
(A) lz-n +i ~i (B) +i ~i (C) zi - (D) -i i - +i ~i
6. The radius of a circle is stated as 2.12 cm. Its area should be written as
(A) 14 cm
2
(B*) 14.1 cm
2
(C) 14.11 cm
2
(D) 14.1124 cm
2
l+=i - +i lzii 2.12 =ni -i: i-i t :=+i -i zi+n <zii i i ni
(A) 14 =ni
2
(B) 14.1 =ni
2
(C) 14.11 =ni
2
(D) 14.1124 =ni
2
Objective - II
1. Asituation may be described by using different sets of co-ordinate axes having different orientations. Which
of the following do not depend on the orientation of the axes ?
(A*) the value of a scalar (B) component of a vector (C*) a vector
(D*) the magnitude of a vector
ll-il- l-< zii + ~-ii + lli-- n +i in +: l-< zi ~-ii + lli-- =n -i ,ii +- +i i-i t l--- n = +i -=i
~-ii + n +i l-i -ti +-i t
(A) l+=i ~l<zi +i ni- (B) l+=i =l<zi +i i+ (C) +i: =l<zi (D) l+=i =l<zi +i lnii
2. Let

C
=

A
+

B
(A) |

C
| is always greater than |

A
| (B*) It is possible to have |

C
| < |

A
| and |

C
|< |

B
|
(C) C is always equal to A + B (D) C is never equal to A+ B
ni-il+ B A C

+ =
(A) | C |

=< | A |

= i ti ni (B) t =-i t l+ | A | | C |

< -ii | B | | C |

< ti
(C) C =< A+ B + -- tini (D) C+ii ii A+ B+ i -ti ti ni
3. Let the angle between two nonzero vectors

A
and

B
be 120o and its resultant be

C
.
(A) C must be equal to | A - B| (B) C must be less than |A - B|
(C*) C must be greater than | A- B | (D) C may be equal to | A- B |
ni-il+ <i ~zi - =l<zii
A

-ii
B

+ n +i i 120 t -ii :=+i liini


C

t
(A) C l-lz-- = |A B| + i tini (B) C l-lz-- = |A B| = +n tini
(C) C l-lz-- = |A B| = ~li+ ti ni (D) C, |A B| + i ti =+-i t
4. The x-component of the resultant of several vectors
(A*) is equal to the sum of the x-component of the vectors
(B*) may be smaller than the sum of the magnitudes of the vectors
(C) may be greater than the sum of the magnitudes of the vectors
(D*) may be equal to the sum of the magnitudes of the vectors.
+z =l<zii + liini +i x-i+
(A) =l<zii + x-l-< zii+i + in + - - ti ni (B) =l<zii + i n + lnii + i n = +n ti =+-i t
(C) =l<zii + i n + lnii + i ti =+-i t (D) =l<zii + i n + lnii + i ti =+-i t
Chapter # 2 Physics & Mathematics
Page # 7
manishkumarphysics.in
5. The magnitudes of the vectors product of two vectors A

and B

may be
(A) greater thanAB (B*) equal to AB (C*) less than AB (D*) equal to zero.
<i =l<zii A

-ii B

+ =l<zi ni- +i lnii ti =+-i t


(A) AB = ~li+ (B) AB + i (C) AB = +n (D) zi - + i
WORKED OUT EXAMPLES
1. A vector has component along the X-axis equal to 25unit and along the Y-axis equal to 60 unit. Find the
magnitude and direction of the vector.
Sol. The given vector is the resultant of two perpendicular vectors, one along the X-axis of magnitude 25 unit and
the other along the Y-axis of magnitude 60 unit. The resultant has a magnitudeAgiven by
90 cos 60 25 2 ) 60 ( ) 25 ( A
2 2
+ + =
=
2 2
) 60 ( ) 25 ( + = 65.
The angle o between this vector and the X-axis is given by
tan o =
25
60
.
2. Find the resulatant of the three vectors shown in figure
3.0m
2.0m
5
.
0
m
Y
X
Sol. Take the axes as shown in the figure,
The x-component of the 5.0 vector = 5.0 m cos 37 = 4.0 m,
the x-component of the 3.0 m vector = 3.0 m and the x-component of the 2.0 m vector = 2.0 m cos 90 = 0.
Hence, the x-component of the resultant
= 4.0 m + 3.0 m + 0 = 7.0 m.
The y-component of the 5.0 m vector = 5.0 m sin 37 = 3.0 m,
the y-component of the 3.0 m + 0 = 7.0 m.
The y-component of the 3.0 vector = 0
and the y-component of the resultant
= 3.0 m + 0 + 2.0 m = 5.0 m.
The magnitude of the resultant vector
=
2
) m 0 . 5 ( ) m 0 . 7 ( +
2
= 8.6 m.
If the angle made by the resultant with the X-axis is u, then
tan u =
component x
component - y

=
0 . 7
0 . 5
or, q = 35.5,
Chapter # 2 Physics & Mathematics
Page # 8
manishkumarphysics.in
3. The sum of the three vectors shown in figure is zero. Find the magnitudes of the vectors OB and OC.
Sol. Take the axes as shown in the figure.
X
Y
B
5m
A
C
45
0
The x-component of
OA
= (OA) cos 90 = 0.
The x-component of
OB
= (OB) cos 0 = 0.
The y-component of
OC
= (OC) cos 135 =
2
1
OC.
Hence, the x-component of the resultant
= OB
2
1
OC. ....(i)
It is given that the resultant is zero and hence is x-component is also zero. From (i),
OB =
2
1
OC. ....(ii)
The y-component of
OA
= OA cos 180 = OA.
The y-component of
OB
= OB cos 90 = 0.
The y-component of
OC
= OC cos 45 =
2
1
OC.
Hence, the y-component of the resultant
=
2
1
OC OA .....(iii)
As the resultant is zero, so is its y-component. From (iiii),
2
1
OC = OA, or OC = 2 OA = 5 2 m.
From (ii), OB =
2
1
OC = 5 m.
4. The magnitude of vectors
OA
,
OB
and
OC
in figure are equal. Find the direction of
OA
+
OB

OC
,
X
Y
B
5m
A
C
45
0
60
30
Sol. Let OA = OB = OC = F.
Chapter # 2 Physics & Mathematics
Page # 9
manishkumarphysics.in
x-component of
OA
= F cos 30 = F
2
3
.
x-component of
OB
= F cos 60 =
2
F
x-component of
OC
= F cos 135 =
2
F
x-component of
OA
+
OB

OC
=
|
|
.
|

\
|
|
.
|

\
|
+
|
|
.
|

\
|
2
F
2
F
2
3 F
=
2
F
(
3
+ 1 +
2
).
y-component of
OA
= F cos 30 =
2
F
.
y-component of
OB
= F cos 150 =
2
3 F
y-component of
OC
= F cos 45 =
2
F
y-component of
OA
+
OB

OC
= |
|
.
|

\
|

|
|
.
|

\
|
+ |
.
|

\
|
2
F
2
3 F
2
F
=
2
F
(1
3

2
).
Angle of
OA
+
OB

OC
with the X-axis
=
) 2 3 1 (
2
F
) 2 3 1 (
2
F
tan
1
+ +

= tan
1
) 2 3 1 (
) 2 3 1 (
+ +

5. Find the resultant of the free vectors
OA
,
OB
and
OC
shown in figure. Radius of the circle is R.
45
45
C
B
A
0
Sol. OA = OC.
OA
+
OC
is along
OB
(bisector) and its magnitude is 2R cos 45 = R/2.
(
OA
+
OC
) +
OB
is along
OB
and its magnitude is
R
2
+ R = R(1 + 2 ).
Chapter # 2 Physics & Mathematics
Page # 10
manishkumarphysics.in
6. The resultant of vectors
OA
and
OB
is perpendicular to
OA
(figure). Find the angleAOB.
u
6m
B
Y
X
4m
0
A
Sol. Take the dotted lines as X, Y axes, x-component of
OA
= 4 m, x-component of
OB
= 6m cos u
x-component of the resultant = (4 + 6 cos u) m
But it is given that the resultant is along Y-axis. Thus, the x-component of the resultant = 0
4 + 6 cos u = 0 or , cosu =
3
2
7. Write the unit vector in the direction of
A

= 5
i

+ j

2
k

.
Sol. |
A

| =
2 2 2
) 2 ( 1 5 + + =
30
.
The required unit vector is
| A |
A

30
5
i

+
30
1
j

30
2
k

.
8. If |
a

+
b

| = |
a

| show that
a

.
Sol. We have |
a

+
b

|
2
= (
a

+
b

).(
a

)
=
a

.
a

+
a

.
b

+
b

.
a

+
b

.
b

= a
2
+ b
2
= 2
a

.
b

.
Similarly,
|
a

|
2
= (
a

).(
a

)
= a
2
+ b
2
2
a

.
b

If |
a

+
b

| = |
a

| ,
a
2
+ b
2
+ 2
a

.
b

= a
2
+ b
2
2
a

.
b

or ,
a

.
b

= 0
or ,
a

.
9. If
a

= 2
i

+ 3 j

+ 4
k

and
b

= 4
i

+ 3 j

+ 2
k

, find the angle beween


a

and
b

.
Sol. Wehave
a

.
b

= ab cosu
or , cosu =
ab
b . a

where u is the angle between


a

and
b

.
Now
a

.
b

= a
x
b
x
+ a
y
b
y
+ a
z
b
z
= 2 4 + 3 3 + 4 2 = 25.
Also a =
2
z
2
y
2
x
a a a + +
=
16 9 4 + +
=
29
Chapter # 2 Physics & Mathematics
Page # 11
manishkumarphysics.in
and b =
2
z
2
y
2
x
b b b + + =
4 9 16 + +
=
29
.
Thus , cosu =
29
25
or , u = cos
1
|
.
|

\
|
29
25
.
10. If
A

= 2
i

+ 7
k

,
B

=
i

+ 2
k

and
C

= j

, find
A

.(
B

).
Sol.
B

= (
i

+ 2
k

) ( j

)
=
i

( j

) + 2
k

( j

)
=
i

+ 2
k

2
k

=
k

+ j

2
i

0 = 2
i

+ j

+
k

.(
B

) = (2
i

3 j

+ 7
k

) . ( 2
i

+ j

+
k

)
= (2) (2) + (3)(1) + (7)(1)
= 0.
11. The volume of a sphere is given by
V =
3
4
tR
3
where R is radius of the sphere. (a) Find the rate of change of volume with respect to R. (b) Find the change
in volume of the sphere as the radius is increased from 20.0 cm to 20.1 cm . Assume that the rate does not
appreciably change between R = 20.0 cm to R = 20.1 cm.
Sol. (a) V =
3
4
tR
3
or ,
dR
dV
=
3
4
t
dr
d
(R)
3
=
3
4
t. 3R
2
= 4tR
2
.
(b) At R = 20 cm , the rate of change of volume with the radius is
dR
dV
= 4tR
2
= 4t (400 cm
2
)
= 1600 t cm
2
.
The change in volume as the radius changes from 20.0 cm to 20.1 cm is
AV =
dR
dV
AR
= (1600 t cm
2
) (0.1 cm)
= 160 t cm
3
.
12. Find the derivative of the following functions with respect to x (a) y = x
2
sinx , (b) y =
x
x sin
and (c) y = sin (x)
2
.
Sol. (a) y = x
2
sinx
dx
dy
= x
2
dx
d
(sinx) + (sinx)
dx
d
(x
2
) ]
= x
2
cosx + (sinx) (2x)
= x (2 sinx + x cosx).
(b) y =
x
x sin
dx
dy
=
2
x
dx
dx
x sin ) x (sin
dx
d
x |
.
|

\
|

Chapter # 2 Physics & Mathematics


Page # 12
manishkumarphysics.in
=
2
x
x sin x cos x
.
(b)
dx
dy
=
2
dx
d
(sinx
2
).
dx
) x ( d
2
= cosx
2
(2x)
= 2x cosx
2
.
13. Find the maximum or minimum values of the function y = x +
x
1
for x > 0.
Sol. y = x +
x
1
dx
dy
=
dx
d
(x) +
dx
d
(x
1
)
= 1 + ( x
2
)
= 1
2
x
1
.
For y to be maximum or minnimum ,
dx
dy
= 0
or , 1
2
x
1
= 0
Thus x = 1 or 1.
For x > 0 the only possible maximum or minimum is at x = 1 , At x = 1 , y = x +
x
1
= 2.
Near x = 0 , y = x +
x
1
is very large because of the term x. Thus , x = 1 must correspond to a minimum. Thus,
y has only a minimum for x > 0. This minimum ocurs at x = 1 and the minimum value of y is y = 2.
14. Figure shows the curve y = x
2
. Find the area of the shaded part between x = 0 and x = 6.
Ax x
Sol. The area can be divided into strips by drawing ordinates between x = 0 and x = 6 at a regular interval of dx.
Consider the strip between the ordinates at x and x + dx. The height of this strip is y = x
2
. The area of this
strip is dA = ydx = x
2
dx.
The total area of the shaded part is obtained by summing up these strips -areas x varying from 0 to 6. Thus,
A =
}
6
0
2
dx x
=
6
0
3
3
x
(
(

=
3
0 216
= 72 .
15. Evaluate
}
e
t
0
t sin A dt where Aand eare constants.
Sol.
}
e
t
0
t sin A dt
Chapter # 2 Physics & Mathematics
Page # 13
manishkumarphysics.in
= A
t
0
t cos
(

e
e
=
e
A
(1 coset).
16. The velocity v and displacement x of a particle executing simple harmonic motion are related as
v
dx
dv
= e
2
x.
At x = 0 , v = v
0
. Find the velocity v when the displacemet becomes x.
Sol. Wehave
v
dx
dv
= e
2
x
or , v dv = e
2
x dx
or , }
v
v
0
dv v
=
}
e
x
0
2
dx x
.....(i)
When summation is made on w
2
x dx the quantity to be varied is x. When summation is made on vdv the
quantityto be varied is v. As x varies from0 to x the velocity varies fromv
0
to v. Therefore , on the left the limits
of integration are from v
0
to v and on the right they are from 0 to x. Simplifying (i) ,
v
v
2
0
v
2
1
(

= e
2
v
v
2
0
v
2
1
(

or ,
2
1
(v
2
v
0
2
) = e
2
2
x
2
or , v
2
= v
0
2
e
2
x
2
or , v =
2 2 2
0
x v e
.
17. The charge flown through a circuit in the time interval between t and t + dt is given by dq = e
t / t
dt where t is
a constant. Find the total charge flown through the circuit between t = 0 to t = t.
Sol. The total charge flown is the sum of all the dq's for t varying from t = 0 to t = t
0
. Thus , the total charge flown
is
Q =
}
t
t
0
/ t
e dt
=
t
t
(
(

t
0
/ t
/ 1
e
= t
|
.
|

\
|

e
1
1
.
18. Evaluate (21.6002 + 234 + 2732.10) 13.
Sol.
21.6002
234
2732.10
22
234
2732
2988
The three numbers are arranged with their decimal points aligned (shown on the left part above). The column
just left to the decimal has 4 as the doubtful digit. Thus , all the numbers are rounded to this column. The
rounded numbers are shown on the rigid part above. The required expression is 2988 13 = 38844. As 13
has only two significant digits the product should be rounded off after two signigicant digits. Thus the result
is 39000.
Chapter # 2 Physics & Mathematics
Page # 14
manishkumarphysics.in
EXERCISE
1. Avector A

makes an angle of 20 and B

makes an angle of 110 with the X-axis. The magnitudes of these


vectors are 3 m and 4m respectively. Find the resultant.
x-~-i = (+ =l<zi
A

, 20 +i i -i-i t -ii <=i =l<zi


B

, 110 +i i -i-i t :- =l<zii + lnii +nzi 3


4 t liini ai- +il
2. Let A

and B

be the two vectors of magnitude of 100 unit and inclined to the X-axis at angles 30 and 60
respectively, find the resultant.
ni-il+ <i =l<zii
A

+ + lnii 10 :+i: t l< :-+i x-~-i = n +i +nzi 30 -ii 60 t . liini


ai- +il
3. Add vectors
A

,
B

and
C

each, having magnitude of 100 unit and inclined to the X-aixs at angles 45, 135
and 315 respectively.
A

,
B

-ii
C

+ +i lnii 100 :+i: -ii :-+ x-~-i = n +i +nzi 45, 135 -ii 315 t . :-+i i n +il
4. Let j

3 i

4 a + =

and j

4 i

3 b + =

(a) Find the magnitudes of


ni-il+ j

3 i

4 a + =

-ii j

4 i

3 b + =

t ai- +il
(A) a

(B) b

(C) b a

+ (D) b a

5. Refer to figure. Find (a) the magnitude, be (b) x and y components and (c) the angle with the resultant of

OA
,

BC
and

DE
.
l-zi + ~iii ai- +il
(a) lnii (b) x -ii y i+
(c)

OA
,

BC
(

DE
+ liini +i x-~-i = +i i
6. Two vectors have magnitudes 3 unit and 4 unit respectively. What should be the angle between them if the
magnitude of the resultant is (a)1 unit, (b) 5 unit and (c) 7 unit
<i =l<zii + lnii +nzi 3 :+i: 4 :+i: t :-+ n +ii l+--i tini l< :-+i liini t
(a) 1 :+i: (b) 5 :+i: (c) 7 :+i:
7. A spy report about a suspected car reads as follows. "The car moved 2.00 km towards east, made a
perpendicular left turn, ran for 500 m, made a perpendicular right turn, ran for 4.00 kmand stopped". Find the
displacement of the car.
(+ i= = = --i < -i t l+ = < t ini +i +i nl-lli := +i t . +i +i ~i 2.00 l+ni. -ni. ii ~i (+
n-- i ni lni. 500 ni. <i i. <ii ~i =n+i i i ni. 4.00 l+ni -ni -ii + ni +i +i l-ii- ai-
+il
8. Acarrom board (4 ft 4ft square) has the queen at the centre. The queen, hit by the striker moves to the front
edge, rebounds and goes in the hole behind the striking line. Find the magnitude of displacement of the
queen (a) from the centre to the front edge, (b) from the front edge to the hole and (c) from the centre to the
hole,
(+ + ni (4 + x 4 + ni +i) n + -< +i- t -i:+ = -i nn- . +i- =in- in l+-i +i ~i
nl-ziin ti -i t . n-i t -ii - i:+ ii + iz l-i- z < n -ni i-i t +i- + l-ii- +i lnii ai- +il
(a) +-< l-< = =in- in l+-i -+
(b) =in- in l+-i = z< -+ -ii
Chapter # 2 Physics & Mathematics
Page # 15
manishkumarphysics.in
(c) +-< = z< -+
Sol.
x 2
4
x
2

=
2 x = 2x
3x = 2
x =
3
2
x 2-x
4
S
1
S
3
S
2
4
>
>
>
1
S

=
3
2
i
+ 2 j

|S
1
| =
4
9
4
+
=
10
3
2
Ans.
|
2
S

| = (2
3
2
)
i
4 j

=
3
4
i
4 j


10
3
4
|
3
S

| =
2 1
S S

+ =
j

4 i

3
4
j

2 i

3
2
+ +
|
3
S

| = 2
i
2 j

|
3
S

| = 2 2
9. A mosquito net over a 7 ft 4 ft bed is 3 ft high. The net has a hole at one corner of the bed through which a
mosquito enters the net. It flies and sits at the diagonally opposite upper corner of the net.(a) Find the
magnitude of the displacement of the mosquito. (b) Taking the hole as the origin, the length of the bed as the
X-axis, its width as the Y-axis, and vertically up as the Z-axis, write the components of the displacement
vector.
7 + x 4 + ~i+i in nn 3 + --i n-z<i-i nni t: t nn + (+ +i- n-z <i-i n (+ z<
t. l== (+ n-z. ini n l ti i-i t t --i t -ii ini + l+i- li- +i- a i-i t
(a) n-z + l-ii- +i lnii ai- +il
(b) z < +i n n l-< . n n +i n-i: +i x-~-i -ii -i i: +i y-~-i ( - i - +i ~i z-~-i ni-- t (. l-ii-
=l<zi + i+ lnli
10. Suppose
a

is a vector of magnitude 4.5 unit due north. What is the vector (a) 3
a

, (b) 4
a

?
ni-il+ =l<zi
a

+i lnii 4.5 :+i: . --i +i ~i t =l<zi +i ti n `


(A)
a 3

(B)
a 4

11. Two vectors have magnitudes 2m and 3m. The angle between themis 60. Find (a) the scalar product of the
two vectors, (b) the magnitude of their vector product.
<i =l<zii + lnii 2 ni. 3 ni. t :-+ n +i i 60 t ai- +il
(a) <i-i =l<zii +i ~l<zi ni- (B) :-+ =l<zi ni- +i lnii
12. Let A
1
, A
2
, A
3
, A
4
, A
5
, A
6
, A
1
be a regular hexagon. Write the x-components of the vectors represented by the
six sides taken in order. Use the fact that the resultant of these six vectors is zero, to prove that cos 0 + cos
3
t
+ cos
3
2t
+ cos
3
3t
+ cos
3
4t
+ cos
3
5t
= 0 . Use the known cosine values to verify the result.
ni-il+ A
1
, A
2
, A
3
, A
4
, A
5
, A
6
, A
1
(+ l-ln- ii t :=+i z i i~i +i +n n i- t (. i i~i = +- =l<zii
+ x-i+ lnli t - +- +- t( :- z =l<zii +i liini zi - t. l=z +il l+
cos 0 + cos
3
t
+ cos
3
2t
+ cos
3
3t
+ cos
3
4t
+ cos
3
5t
= 0
Chapter # 2 Physics & Mathematics
Page # 16
manishkumarphysics.in
liin +i +i i~i + ni- +i -in ++ =l-lz-- +il
13. Let k

4 j

3 i

2 a + + =

and k

5 j

4 i

3 b + + =

. Find the angle between them.


ni-il+ k

4 j

3 i

2 a + + =

-ii k

5 j

4 i

3 b + + =

:-+ n +ii ai- +il


14. Prove that 0 ) B x A ( . A =

l=z +il l+ 0 ) B x A ( . A =

15. If k

4 j

3 i

2 A + + =

and k

2 j

3 i

4 B + + =

, find B x A

.
l< k

4 j

3 i

2 A + + =

-ii k

2 j

3 i

4 B + + =

ai- +il B x A

16. If C , B , A

are mutually perpendicular, showthat 0 ) B x A ( x C =

. Is the converse true?
l< C , B , A

- n-- t . +- +il l+ 0 ) B x A ( x C =

+i :=+i +n ii = t `
17. A particle moves on a given straight line with a constant speed u. At a certain time it is at a point P on its
straight line path. O is a fixed point. Show that v x OP

is independent of the position?


(+ +i <i n: =n ii + ~- l<zi l-- -in v = nl-ziin t l+=i l-lz-- =n t t =n ii i + l-<
P t O(+ l-i l-< t +- +il l+ v x OP

, P+i l-il- l-i -ti +-i t


18. The force on a charged particle due to electric and magnetic fields is given by B x v q E q F


+ = . Suppose E

is along the X-axis and


B

along the Y-axis. In what direction and with what minimum speed u should a
positively charged particle be sent so that the net force on it is zero?
l+=i ~i lzi- +i l - ( - -+i -i zii + +ii n. B x v q E q F


+ = ,ii +- l+i i-i t ni-il+ E

. x-
~-i + ~- l<zi t -ii
B

, y-~-i + ~- l<zi t (+ i-i lzi+ +i +i l+= l<zii n -ii l+= - --n -in v = nl- +i:
i l+ := liini n zi - ti `
19. Give an example for which B . C B . A

= but
C A

=
B . C B . A

= l+--
C A

=
+i (+ -<iti <il
20. Drawa graph from the following data. Drawtangents at x = 2, 4, 6 and 8. Find the slopes of these tangents.
Verify that the curve drawn is y = 2x
2
and the slope of tangent is tanu =
dx
dy
= 4x.
l---lnli- ~i+i +i -in ++ niil-zi -i: x = 2, 4, 6 -ii 8 -zi ii( ii-i :- -zi ii~i +
cin ai- +il l=z +il l+ ii-i ni + y = 2x
2
t -ii -zi ii +i cin tan u =
dx
dy
= 4x t
x 1 2 3 4 5 6 7 8 9 10
y 2 8 18 32 50 72 98 128 162 200
21. A curve is represented by y = sin x. If x is changed from (+ + y = sin x
3
t
to
3
t
+
100
t
, find approximately
the change in y
Chapter # 2 Physics & Mathematics
Page # 17
manishkumarphysics.in
,ii +- l+i i-i t l< x +i ni- ll- - ++
3
t
=
3
t
100
t
+ l<i i-i t . y + ni- n ~- nil-- l- -
ai- +il
22. The electric current in a charging R-Ccircuit is given by i = i
o
t/RC
where i
o
, Rand C are constant parameters
of the circuit and t is time. Find the rate of change of current in at (a) t = 0, (b) t = RC, (c) t = 10 RC.
R-Cli n ~i zi- + =n iii i = i
0
e
t/RC
,ii +- +i i-i t . ti i
0
, R-ii Cli +i l-- ilzii t -ii
t =n t =n (a) t = 0, (b) t = RC, (c) t = 10 RC li n iii + ni- n l-- +i < ai- +il
23. The electric current in a discharging R-C circuit is given by R-C i = i
0
e
t/RC
where i
o
, R and C are constant
parameters and t is time. Let i
o
= 2.00A, R = 6.00 10
5
O and C = 0.500 F. (a) Find the current at t = 0.3s.
(b) Find the rate of change of current at t = 0.3 s. (c) Find approximately the current at t = 0.31 s.
li + ln( l= - +in n l - iii i = i
0
e
t/RC
,ii +- +i i-i t . ti i
0
, R-ii C li +i l-- ilzii
t -ii t =n t ni-il+ i
0
= 2.00A, R = 6.00 x 10
5
O -ii C = 0.500 F
(a) t = 0.3 = iii ai- +il
(b) t = 0.3 = iii n l-- +i < ai- +il
(c) t = 0.31 =. iii +i ni- (nnin) ai- +il
24. Find the area bounded under the curve y = 3x
2
+ 6x + 7 and the X-axis with the ordinates at x = 5 and x =
10.
+ y = 3x
2
+ 6x + 7 -ii l-<zii+i x = 5 -ii x = 10 + i- x-~-i ,ii lz -izi+n ai- +il
25. Find the area enclosed by the curve y = sin x and the X-axis between x = 0 and x = t.
+ y = sin x -ii x = 0 ( x = t + i- x-~-i ,ii lz -izi+n ai- +il
26. Find the area bounded by the curve y = e
x
, the X-axis and the Y-axis
+ y = e
x
, x-~-i -ii y-~-i ,ii lz -i zi+n ai- +il
27. Arod of length L is placed along the X-axis between x = 0 and x = L. The linear density (mass/length) p of the
rod varies with the distance x from the origin as p = a + bx. (a) Find the SI units of a and b. (b) Find the mass
of the rod in terms of a,b,and L.
L n-i: +i (+ z x-~-i + ~- l<zi x = 0 -ii x = L + n ii t : t z +i li+ i- (< ni-n-i: ) P
nn l-< = <i x + =ii P = a + bx + ~-=i ll-- ti-i t
(a) a -ii b +i SI :+i:i ai- +il
(b) a, b -ii L + <i n z +i <ni- ai- +il
28. The momentum p of a particle changes with time t according to the relation
dt
dp
= (10N) + (2N/s)t. If the
momentum is zero at t = 0, what will the momentum be at t = 10s ?
l+=i +i +i = n. =n + =i -i = zi
dt
dp
= (10 --) + (2 - -= .)t + ~- =i ll-- ti-i t l< t + ~- =i
ll-- ti -i t l< t = 0 =n zi- ti . -i t = 10 =. =n l+--i tini `
29. The changes in a function y and the independent variable x are related as dy/dx = x
2
. Find y as a function of
x.
+n- y n l-- -ii --zi - x n =--i dy/dx = x
2
t x + +n- + n y ai- +il
30. Write the number of significant digits in
=ii+ =i( lnli
(a) 1001 (b) 100.1 (c) 100.10 (d) 0.001001
Chapter # 2 Physics & Mathematics
Page # 18
manishkumarphysics.in
31. Ametre scale is graduated at every millimetre. Howmany significant digits will be there in a length measure-
ment with this scale?
(+ ni ni-i + lnnini n liil- t := ni- ,ii nii n: n-i: n l+--i =ii+ =i( t `
32. Round the following numbers to 2 significant digits.
l---lnli- =i~i +i 2 =ii+ ~+i -+ =iln- +il
(a) 3472 (b) 84.16 (c) 2.55 (d) 28.5
33. The length and the radius of a cylinder measured with a slide callipers are found to be 4.54 cm and 1.75 cm
respectively. Calculate the volume of the cylinder.
-ni: + ln= +i =ti-i = (+ n- +i n-i: -ii lzii nii i-i t . :-+ ni- +nzi 4.54 = ni ( 1.75 = ni
i-- ti- t n- +i ~i-- +i ni-i +il
34. The thickness of a glass plate is measured to be 2.17 mm, 2.17 mm and 2.18 mmat three different places.
Find the average thickness of the plate from this data.
(+ +i - +i l +i +i -i- li--li-- -ii-i nii ni ni i: i 2.17 lnni. 2.17 lnni -ii 2.18 lnni t :- ~i + i
+i =ti-i = l+i +i ~i=- nii: ai- +il
35. The length of the string of a simple pendulum is measured with a metre scale to be 90.0 cm. The radius be
2.13 cm using measurements with a slide callipers. What is the effective length of the pendulum? (The
effective length is length of the pendulum between the point of suspension and the centre of the bob.)(+ ni
ni- +i =ti-i = =n ni n+ +i i i +i n-i: 90.0 = ni nii i-i t -ni: + ln= = ni- i ( t +
+i =l-nln- n-i: 2.13 = ni i-- ti -i t ni n+ +i iii n-i: l+--i t ` (l-n-- l-< -ii i + + -< + i-
+i < i iii n-i: +tni-i t )

Вам также может понравиться